Difference between revisions of "2023 AMC 12B Problems/Problem 2"

(Blanked the page)
(Tag: Blanking)
Line 1: Line 1:
 +
==Problem==
  
 +
Carlos went to a sports store to buy running shoes. Running shoes were on sale, with prices reduced by <math>20\%</math> on every pair of shoes. Carlos also knew that he had to pay a <math>7.5\%</math> sales tax on the discounted price. He had <math>\$43</math> dollars. What is the original (before discount) price of the most expensive shoes he could afford to buy?
 +
 +
 +
<math>\textbf{(A) }\$46\qquad\textbf{(B) }\$50\qquad\textbf{(C) }\$48\qquad\textbf{(D) }\$47\qquad\textbf{(E) }\$49 </math>

Revision as of 18:36, 15 November 2023

Problem

Carlos went to a sports store to buy running shoes. Running shoes were on sale, with prices reduced by $20\%$ on every pair of shoes. Carlos also knew that he had to pay a $7.5\%$ sales tax on the discounted price. He had $$43$ dollars. What is the original (before discount) price of the most expensive shoes he could afford to buy?


$\textbf{(A) }$46\qquad\textbf{(B) }$50\qquad\textbf{(C) }$48\qquad\textbf{(D) }$47\qquad\textbf{(E) }$49$